Complex Analysis: Integration

Click For Summary
The integral to evaluate is expressed in terms of cosine using complex exponentials, transforming it into a contour integral over the unit circle. The integral simplifies to a form involving a quadratic equation in z, where the poles depend on the value of r. A suggestion is made to leave r in the equation to accurately determine the poles by solving the quadratic. Additionally, there are indications of algebraic errors in the last steps of the attempted solution, prompting a review of those calculations. Correctly identifying the poles is crucial for further evaluation of the integral.
Shay10825
Messages
337
Reaction score
0

Homework Statement



Evaluate the following integral for 0<r<1 by writing \cos\theta = \frac{1}{2}(e^{i\theta} + e^{-i\theta}) reducing the given integral to a complex integral over the unit circle.

Evaluate: \displaystyle{\frac{1}{2\pi}\int_0^{2\pi}\frac{1}{1-2r\cos\theta + r^2}\,d\theta}


Homework Equations



none

The Attempt at a Solution



\displaystyle\cos\theta = \frac{1}{2}(e^{i\theta} + e^{-i\theta})}

\displaystyle{z=e^{i\theta}}

\displaystyle{ \cos t= \frac{1}{2}(z+ \frac{1}{z})}

\displaystyle{\frac{dz}{iz}= dt}

~~~~~~~~~~~~~~~~~~~~~~~~~~~~~~~

\displaystyle{\frac{1}{2\pi}\oint \frac{1}{1-2r[\frac{1}{2}(z+\frac{1}{z})]+r^2}\,\frac{dz}{iz}}

\displaystyle{\frac{1}{2\pi}\oint \frac{1}{z-2rz[\frac{1}{2}(z+\frac{1}{z})]+r^2}\,\frac{dz}{i}}

\displaystyle{\frac{-i}{2\pi}\oint \frac{1}{z-2rz[\frac{1}{2}(z+\frac{1}{z})]+r^2}\,dz}

\displaystyle{\frac{-i}{2\pi}\oint \frac{1}{z-r^2z^2-r+r^2}\,dz}

But I get stuck here. What do I do with the "r"? Should I factor it out, and if yes then how?

Thanks
 
Physics news on Phys.org
Shay10825 said:
\displaystyle{\frac{-i}{2\pi}\oint \frac{1}{z-r^2z^2-r+r^2}\,dz}

But I get stuck here. What do I do with the "r"? Should I factor it out, and if yes then how?

Thanks

Leave the "r" where it is, the location of the poles will depend on its value...find those poles by solving the quadratic z-r^2z^2-r+r^2=0 for z...

Edit: You've also got a couple of algebra errors, double check your last 4 steps
 
Last edited:
Question: A clock's minute hand has length 4 and its hour hand has length 3. What is the distance between the tips at the moment when it is increasing most rapidly?(Putnam Exam Question) Answer: Making assumption that both the hands moves at constant angular velocities, the answer is ## \sqrt{7} .## But don't you think this assumption is somewhat doubtful and wrong?

Similar threads

Replies
13
Views
2K
  • · Replies 34 ·
2
Replies
34
Views
3K
Replies
3
Views
2K
  • · Replies 19 ·
Replies
19
Views
2K
  • · Replies 4 ·
Replies
4
Views
1K
  • · Replies 3 ·
Replies
3
Views
2K
  • · Replies 6 ·
Replies
6
Views
2K
  • · Replies 14 ·
Replies
14
Views
2K
  • · Replies 8 ·
Replies
8
Views
2K
  • · Replies 4 ·
Replies
4
Views
2K